Đến nội dung

Secrets In Inequalities VP nội dung

Có 298 mục bởi Secrets In Inequalities VP (Tìm giới hạn từ 30-03-2020)



Sắp theo                Sắp xếp  

#410835 Chứng minh $d$ chia hết cho $2010$.

Đã gửi bởi Secrets In Inequalities VP on 06-04-2013 - 19:57 trong Số học

Cho hai đa thức hệ số nguyên $P(x)$ và $Q(x)$ thỏa mãn $P(x^3)+xQ(x^3)$ chia hết cho $x^2+x+1$. Gọi $d$ là ước chung lớn nhất của$P(2011)$ và $Q(2011)$. Chứng minh rằng $d$ chia hết cho $2010$

Sử dụng $P(a)-P(b)\vdots a-b$ ta được : $P(x^3)-P(1)\vdots x^3-1\vdots x^2+x+1$

và $xQ(x^3)-xQ(1)\vdots x^3-1\vdots x^2+x+1$

$\Rightarrow P(1)+xq(1)\vdots x^2+x+1$.Với $x$ đủ lớn thì $VP>VT$ $\Rightarrow P(1)+xQ(1)= 0\Rightarrow P(1)=Q(1)=0$

$\Rightarrow P(x)= (x-1)R(x);Q(x)= (x-1)H(x);$.Dễ chứng minh $R(x),H(x)\in\mathbb{Z}[x]$

$\Rightarrow P(2011)= 2010R(x);Q(2011)= 2010H(x);$

$\Rightarrow d\vdots 2010$




#409878 1/Tìm GTLN của $|a^3+b^3+c^3-abc|$ biết $a^2+b^2+c^2=1$.

Đã gửi bởi Secrets In Inequalities VP on 02-04-2013 - 11:29 trong Bất đẳng thức - Cực trị

Bài 1: Cho $a,b,c \in \mathbb{R}$ thỏa mãn $a^2+b^2+c^2=1$.

Tìm GTLN của  $P=\left | a^3+b^3+c^3-abc \right |$

 

Bài 2:  Cho $a,b,c \in \mathbb{R}$ thỏa mãn $\left | a^3+b^3+c^3-abc \right |=1$.

Tìm GTNN của $a^2+b^2+c^2$

 

Để ý $(a^3+b^3 )^{2}\leq (a^2+b^2)(a^4+b^4)$

Ta có : $$P^2= [a^3+b^3+c(c^2-ab)]^{2}\leq (a^2+b^2+c^2)(a^4+b^4+(c^2-ab)^2)$$

$$= a^4+b^4+c^4+a^2b^2-2abc^2\leq a^4+b^4+c^4+a^2b^2+b^2c^2+c^2a^2$$

$$\leq a^4+b^4+c^4+2a^2b^2+2b^2c^2+2c^2a^2= (a^2+b^2+c^2)^{2}$$

 
Đến đây xong cả hai bài !



#408291 $f(mf(n))=n^2.f(m), \forall m,n \in \mathbb{N}^...

Đã gửi bởi Secrets In Inequalities VP on 27-03-2013 - 12:22 trong Phương trình hàm

http://diendantoanho...1-số-nguyen-tố/




#405539 Tìm MAX T

Đã gửi bởi Secrets In Inequalities VP on 16-03-2013 - 17:08 trong Tổ hợp và rời rạc

Cho bảng ô vuông 3$\times$3.Người ta điền tất cả các số từ 1$\rightarrow$9 vào các ô vuông của bảng sao cho 4 ô vuông của bảng tạo thành 1 hình vuông kích thuước 2$\times$2 và tổng 4 số trên hình vương mới được tạo thành đều bằng nhau và bằng T.Tìm MAX T

Đánh số các ô theo hàng ngang từ trên xuông dưới là $a,b,c,d,e,f,g,h,i$ thì ta có :
$4T= a+c+g+i+2(b+d+f+h)+4e= 1+2+...+9+3e+b+d+f+h$
$\leq 46+3.9+8+7+6+5= 98\Rightarrow T\leq 24$
Vậy $T max$ bằng $24$ có thể thấy ngay có một cách điền thõa mãn là :
$a=1,b=9,c=2,d=6,e=8,f=5,g=3,h=7,i=4$

p/s : Làm bài thế nào em ?



#398581 $\begin{matrix} f:\mathbb{N}\rightarrow \mathbb{N}...

Đã gửi bởi Secrets In Inequalities VP on 20-02-2013 - 18:22 trong Phương trình hàm

$\begin{matrix} f:\mathbb{N}\rightarrow \mathbb{N}\\ mf(n)+nf(m)=(m+n)f(m^{2}+n^{2})\forall m,n\epsilon \mathbb{N} \end{matrix}$

Ta thấy ngay hàm hằng $f(x)=c$ thỏa mãn.Giả sử $f$ k là hàm hằng.
Chọn $m$ và $n$ là $2$ số sao cho $f(m)-f(n)>0$ và nhỏ nhất.
$\Rightarrow f(n)= \frac{nf(n)+mf(n)}{m+n}< \frac{nf(m)+mf(n)}{m+n}< \frac{nf(m)+mf(m)}{m+n}= f(m)$
$\Rightarrow f(n)< f(m^2+n^2)< f(m)$
$\Rightarrow 0<f(m^2+n^2)-f(n)< f(m)-f(n)$ .Vô lí theo cách chọn $m,n$



#396749 Chứng minh rằng với mọi số nguyên tố $p$ thì $f(p)$ là mộ...

Đã gửi bởi Secrets In Inequalities VP on 14-02-2013 - 22:32 trong Phương trình hàm

Bài này phải là $f: \mathbb{N}^* \to \mathbb{N}^*$, chứ hàm $f \equiv 0$ thỏa hệ thức nhưng không thỏa đề.

====
Chắc đề in thiếu ^_^

Cố định $m$ dễ thấy $f$ đơn ánh.
Cho $m=n=1$ ta được $f(f(1))= f(1)\Rightarrow f(1)= 1$
Ta chứng minh $f$ nhân tính :
Cho $m=1;n\in\mathbb{N}^{*}$ ta được $f(f(n))= n^{2}$
Thay $m$ bởi $f(m)$ trong đề bài ta được:
$f(f(m).f(n))= n^{2}.f(f(m))= m^{2}n^{2}= (mn)^2.f(1)= f(f(mn))$
$\Rightarrow f(m)f(n)=f(mn)$
Giả sử kết luận bài toán là sai suy ra tồn tại $a\neq b\in\mathbb{N}^{*},a> 1,b> 1$ sao cho $f(p)=ab$
$\Rightarrow p^2= f(f(p))=f(ab)=f(a).f(b)$
Do $f$ đơn ánh và $a> 1,b> 1\Rightarrow f(a),f(b)> f(1)=1$
Chỉ xảy ra các TH sau :
+ TH1: $f(a)\vdots p^{2},f(b)\not\vdots p\Rightarrow f(a)\geq p^{2}\Rightarrow f(a)f(b)> p^2$ .Vô lí
+ TH2: $f(b)\vdots p^{2},f(a)\not\vdots p$ Vô lí.
+ TH3: $f(a)\vdots p,f(b)\vdots p\Rightarrow f(a)\geq p;f(b)\geq p\Rightarrow p^{2}= f(a)f(b)\geq p^{2}$
$\Rightarrow f(a)= f(b)= p\Rightarrow a= b$ .Vô lí
Vậy giả sử sai và ta có đpcm



#393868 Đề thi HSG lớp 10 trường THPT Chuyên Hà Nội-Amsterdam

Đã gửi bởi Secrets In Inequalities VP on 06-02-2013 - 17:47 trong Thi HSG cấp Tỉnh, Thành phố. Olympic 30-4. Đề thi và kiểm tra đội tuyển các cấp.

Đề thi HSG lớp 10 trường THPT Chuyên Hà Nội-Amsterdam
Thời gian: 90'
Bài 1:
Cho $a,n\in \mathbb{Z}^+$ thỏa mãn $(a+1)^3-a^3=n^2$
Chứng minh rằng $n$ là tổng của hai số chính phương liên tiếp


Xét pt $Pell$ : $x^{2}-3y^{2}=1$...



#393855 $y_{n}^{2}=3x_{n}^{2}+1,\fo...

Đã gửi bởi Secrets In Inequalities VP on 06-02-2013 - 17:33 trong Dãy số - Giới hạn

Giải:
Ta có số hạng tổng quát của dãy $(x_n)$ là $x_n=(\frac{3-\sqrt{3}}{6})(2+\sqrt{3})^n+(\frac{3+\sqrt{3}}{6})(2-\sqrt{3})^n$
Tương tự, số hạng tổng quát của dãy $(y_n)$ là $y_n=\frac{1}{2}((2+\sqrt{3})^n+(2-\sqrt{3})^n)$
Đến đây, ta chỉ việc thay vào và suy ra $y_n^2=3x_n^2+1,\forall n\in \mathbb{N}$. ~O)

$2$ dãy trên là dãy nghiệm của pt $Pell$ : $a^{2}-3b^{2}=1$ từ đó dễ suy ra đpcm



#393852 $a_{n}$ là số nguyên dương,$\forall n\in...

Đã gửi bởi Secrets In Inequalities VP on 06-02-2013 - 17:26 trong Dãy số - Giới hạn

Cho dãy số $(a_{n})$:$(a_{n})\left\{\begin{matrix} a_{0}=1\\ a_{n}=(7a_{n-1}+\sqrt{45a_{n-1}^{2}-36})\div 2,\forall n\in \mathbb{N}^{*} \end{matrix}\right.$
Chứng minh:
a)$a_{n}$ là số nguyên dương,$\forall n\in \mathbb{N}$
b)$a_{n+1}a_{n}-1$ là số chính phương,$\forall n\in \mathbb{N}$

a) Bằng qui nạp dễ cm $({a_n})$ là dãy tăng.
Từ hệ thức ban đầu ta suy ra : $$(2a_{n}-7a_{n-1})^{2}= 45a_{n-1}-36$$
Rút gọn ta được : $$2a_{n}^{2}-7a_{n}a_{n-1}+2a_{n-1}^{2}+18=0$$
Thay $n$ bởi $n+1$ :$$2a_{n+1}^{2}-7a_{n+1}a_{n}+2a_{n}^{2}+18=0$$
Trừ từng vế $$\Rightarrow 2a_{n+1}^{2}-2a_{n-1}^{2}+7a_{n}(a_{n-1}-a_{n+1})= 0$$
$$\Rightarrow (a_{n+1}-a_{n-1})(2a_{n+1}-7a_{n}+2a_{n-1})= 0$$
Vì $({a_n})$ là dãy tăng nên $a_{n+1}-a_{n-1}> 0\Rightarrow 2a_{n+1}-7a_{n}+2a_{n-1}=0$
Từ hệ thức này kết hợp qui nạp dễ thấy $Q.E.D$

b) Ta có : $a_{0}=1;a_{1}=5;a_{2}=34;2a_{n+1}-7a_{n}+2a_{n-1}=0$
Qui nạp một tí là ra : $a_{n+1}a_{n-1}-a_{n}^{2}=9$
Thêm bớt thành hằng đẳng thức tí nữa là ra .



#393824 $n=d_{1}^{2}+d_{2}^{2}+d_{3...

Đã gửi bởi Secrets In Inequalities VP on 06-02-2013 - 16:36 trong Số học

Let $1=d_{1}<d_{2}<...<d_{k}=n$ be all different divisors ò positive integer n written in ascending order. Determine all n such that:
a)$n=d_{1}^{2}+d_{2}^{2}+d_{3}^{2}+d_{4}^{2}$.
b)$n+1=d_{6}^{2}+d_{7}^{2}$.
c)$n^{2}= (d_{7}^{2}+d_{10}^{2})d_{22}^{2}$.

Nếu $n$ lẻ suy ra tất cả các ${d_i}$ đều lẻ suy ra $n=d_{1}^{2}+d_{2}^{2}+d_{3}^{2}+d_{4}^{2}$ chẵn vô lí
$\Rightarrow n$ chẵn $\Rightarrow {d_2}= 2$$\Rightarrow n= 5+d_{3}^{2}+d_{4}^{2}$
$\Rightarrow n\equiv 1+d_{3}^{2}+d_{4}^{2}(Mod4)$
Mà $d_{3}^{2}+d_{4}^{2}$ chỉ đồng dư vs $0,1,2$ mod $4$ suy ra $\Rightarrow n\equiv 1,2,3(Mod4)$
Mà $n$ chẵn $\Rightarrow n\equiv 2(Mod4)$
Suy ra $({d_1},{d_2},{d_3},{d_4})= (1,2,p,q)$ hoặc $(1,2,p,2p)$ trong đó $p,q$ ntố lẻ.
Nếu mà $({d_1},{d_2},{d_3},{d_4})= (1,2,p,q)$ $\Rightarrow n= 3(Mod4)\Rightarrow VL$
$\Rightarrow ({d_1},{d_2},{d_3},{d_4})= (1,2,p,2p)$
$\Rightarrow n= 5+p^{2}+4p^{2}= 5(1+p^2)\vdots 5\Rightarrow p= 5\Rightarrow n= 130$



#393460 Lập team và chiến dota chào xuân 2013 nào!

Đã gửi bởi Secrets In Inequalities VP on 05-02-2013 - 19:37 trong Góc giao lưu

Em một slot a ơi ! :))



#388497 $a^{2} + b^{2} - abc$ $=$ $z^...

Đã gửi bởi Secrets In Inequalities VP on 20-01-2013 - 13:39 trong Số học

Cho $\text{a , b , c} \in \mathbb{N}$. Chứng minh rằng :
Nếu $0 < a^{2} + b^{2} - abc \leq c$ thì $a^{2} + b^{2} - abc$ là một số chính phương.

Ý tưởng là Vieta-jumping thôi.
Đặt $a^{2} + b^{2} - abc= k\leq c$
Giả sử có ${a_0},{b_0},{c_0}$ thỏa đề và ${a_0}+{b_0}$ là nhỏ nhất trong đó ${a_0}\geq {b_0}$
+ Nếu ${a_0}={b_0}$ thì dễ thấy đpcm.
+ Nếu ${a_0}> {b_0}$ thì viết lại pt đầu dưới dạng pt bậc $2$ ẩn ${a_0}$ thì nó sẽ có thêm $1$ nghiệm $a$ nữa thỏa mãn
$a+{a_0}= bc;a{a_0}= b^{2}-k$
$a< 0\Rightarrow k-b^{2}\geq bc$ vô lí vì $k\leq c$
$a> 0\Rightarrow b^{2}-k-bc+1= (a-1)({a_0}-1)\geq b^{2}$ vô lí
$\Rightarrow a= 0\Rightarrow k= b^{2}$ đpcm



#387273 Tìm GTNN của $\sum \sqrt{\frac{xy}{x^{2}+y^{2}}}$

Đã gửi bởi Secrets In Inequalities VP on 16-01-2013 - 22:07 trong Bất đẳng thức và cực trị

Cho x, y, z là 3 số thực không âm, biết không có 2 số nào đồng thời bằng 0 và $x^{2} + y^{2}+z^{2}=2(xy+yz+zx)$. Tìm GTNN của $\sqrt{\frac{xy}{x^{2}+y^{2}}} + \sqrt{\frac{yz}{y^{2}+z^{2}}} +\sqrt{\frac{zx}{z^{2}+x^{2}}}$

Lâu lâu làm tí BĐT cho vui ! :)
$\sqrt{\frac{xy}{x^{2}+y^{2}}}= \frac{xy}{\sqrt{xy(x^2+y^2)}}\geq \frac{\sqrt{2}xy}{x^{2}+y^{2}}\geq \frac{\sqrt{2}xy}{x^{2}+y^{2}+z^{2}}$
Tương tự rồi cộng vào áp dụng giả thiết được Min là $\frac{\sqrt{2}}{2}$
__________
Cái BĐT cuối cùng,dấu "=" khi $z=0$ nhưng nếu tương tự thì $x=y=z=0$,mâu thuẫn đề bài!
__________
p/s@Tùng : Cậu xem lại nhé.Chỉ có cái đầu là $z=0$ thôi.Dấu "=" là $z=0,x=y> 0$ .



#387270 $\left ( a+b \right )^{6}+ (b+c)^{6} + (c+...

Đã gửi bởi Secrets In Inequalities VP on 16-01-2013 - 22:03 trong Bất đẳng thức và cực trị

2) cho a,b,c không âm thoả ab+bc+ca=1
$\frac{1}{a+b}+\frac{1}{b+c}+\frac{1}{c+a}\geq \frac{5}{2}$

Bài này ngày trước mình đã làm ở đây : http://diendantoanho...b-ck-frac1c-ak/

Bình phương và áp dụng IRan 96
$VT^2= \sum \frac{1}{(a+b)^2}+2\sum \frac{1}{(a+b)(a+c)}= \sum \frac{1}{(a+b)^2}+\frac{4(a+b+c)}{(a+b)(b+c)(c+a)}$
$\geq \frac{9}{4(ab+bc+ca)}+\frac{4(a+b+c)}{(a+b)(b+c)(c+a)}= \frac{9}{4}+\frac{4(a+b+c)(ab+bc+ca)}{(a+b)(b+c)(c+a)}$
$= \frac{9}{4}+\frac{4(a+b)(b+c)(c+a)+4abc}{(a+b)(b+c)(c+a)}= \frac{9}{4}+4+\frac{4abc}{(a+b)(b+c)(c+a)}$
( Vì $(a+b+c)(ab+bc+ca)=(a+b)(b+c)(c+a)+abc$ )
$\geq \frac{9}{4}+4= \frac{25}{4}\Rightarrow VT\geq \frac{5}{2}$




#386809 $3^{a} = 2b^2 +1$

Đã gửi bởi Secrets In Inequalities VP on 14-01-2013 - 22:14 trong Số học

Giải phương trình nghiệm nguyên dương :

$3^{a} = 2b^2 +1$

Các bước chính để giải bài này :
+ Xét $a$ chẵn : Dễ
+ Xét $a$ lẻ .$a=2k+1$.Đặt $x= 3^{k},y= n$ được pt $Pell$ $3x^{2}-2y^{2}= 1$
Giải ra rồi xét Mod $27$ và $17$ là ra ngay.
Lời giải chi tiết mình sẽ post sau :)

@supermember: cám ơn bạn vì đã chú ý 1 bài Toán khá cũ còn tồn đọng như thế này :)

VMF cần có nhiều người như bạn để tránh lãng phí những bài hay :)



#386024 Bên lề VMO 2013

Đã gửi bởi Secrets In Inequalities VP on 12-01-2013 - 19:55 trong Thi HSG Quốc gia và Quốc tế

Bài $1$ là bất đẳng thức trá hình rồi đấy chị :))



#385745 Chứng minh rằng tồn tại $1$ số chia hết cho $2^k$ mà khi...

Đã gửi bởi Secrets In Inequalities VP on 11-01-2013 - 22:09 trong Số học

Bài toán : Chứng minh rằng tồn tại $1$ số chia hết cho $2^k$ mà khi viết nó trong hệ thập phân không chứa số $0$ nào với mọi $k$ nguyên dương.

Bài này cho vào box Số học thíc hợp hơn !
Ta sẽ cm bằng qui nạp :
+ $k=1$ ta có số $2$ thỏa mãn.
+ Giả sử kết luận của bài toán đúng tới $k$, ta cm nó đúng với $k+1$.
Theo gtqn tồn tại số $A\vdots 2^{k}$ có $k$ c/số tm đề bài.
Nếu $A\vdots 2^{k+1}$ thì thêm c/số $2$ vào trc $A$ ta đc số $2.10^{k}+A= 2^{k+1}.5^{k}+A\vdots 2^{k+1}$ và số này thỏa đề
Nếu $A\vdots 2^{k};A\not\vdots 2^{k+1}$ thì viết $A= 2^{k}.p$ trong đó $p$ lẻ.
Xét số có được sau khi thêm c/số $1$ vào trc số $A$ : $10^{k}+A= 2^{k}.(5^{k}+p)\vdots 2^{k+1}$ do $p$ lẻ và do đó số này thỏa đề.
Vậy theo nguyên lí qui nạp ta có $Q.E.D$



#385626 $S = \sum\limits_{k = m}^n {\frac{1...

Đã gửi bởi Secrets In Inequalities VP on 11-01-2013 - 19:08 trong Số học

Chứng minh rằng tổng: $S = \sum\limits_{k = m}^n {\frac{1}{k}} \left(m,n \in\mathbb N^*\right)$ không là số nguyên. $(n \ge m+1)$
___
NLT

Chết thật ! Lúc đấy mắt mũi cà là té hay sao ấy ! :biggrin: Fix lại như sau , ý tưởng vẫn như lời giải sai ở trên :
Đặt $n= m+i$.
Kí hiệu $v(x)$ là số mũ lớn nhất của $2$ trong $x$.
Giả sử $S$ là số nguyên .
Đặt $K= lcm[m,m+1,m+2,...,m+i]$ suy ra $K$ là số chẵn.
Do đó : $KS= \frac{K}{m}+\frac{K}{m+1}+...+\frac{K}{m+i}$ là số chẵn.
Đặt ${a_k}= v(k)$ với mọi $k=m,m+1,m+2,...,m+i$.
Gọi ${a_s}= max({a_k})$ $\Rightarrow v(K)= {a_s}$
Giả sử ngoài $s$ ra còn có số $t$ sao cho ${a_t}= {a_s}$ thì viết $t=2^{{a_t}}.{t_1};s=2^{{a_s}}.{s_1}$ trong đó ${t_1},{s_1}$ lẻ.
Suy ra tồn tại số chẵn ${r_1}$ nằm giữa ${s_1}$ và ${t_1}$ $\Rightarrow r= 2^{{a_s}}.{r_1}<t<K\Rightarrow r|K$
Nhưng $r= 2^{{a_s}}.{r_1}\vdots 2^{{a_s}+1}>2^{{a_s}}= v(K)$ suy ra vô lí.
Vậy tồn tại duy nhất số $s$ sao cho ${a_s}= max({a_k})$.Suy ra trong các số $\frac{K}{k}$ chỉ có $\frac{K}{s}$ chẵn.
Suy ra $KS= \frac{K}{m}+\frac{K}{m+1}+...+\frac{K}{m+i}$ là lẻ.Mâu thuẫn với ở trên.
$\Rightarrow Q.E.D$



#385581 $S = \sum\limits_{k = m}^n {\frac{1...

Đã gửi bởi Secrets In Inequalities VP on 11-01-2013 - 16:15 trong Số học

Chứng minh rằng tổng: $S = \sum\limits_{k = m}^n {\frac{1}{k}} \left(m,n \in\mathbb N^*\right)$ không là số nguyên.

Kí hiệu $v(x)$ là số mũ lớn nhất của $2$ trong $x$.
Giả sử $S$ là số nguyên .
Đặt $m= lcm[1,2,...,n]$ suy ra $m$ là số chẵn.
Do đó : $mS= \frac{m}{1}+\frac{m}{2}+...+\frac{m}{n}$ là số chẵn.
Đặt ${a_k}= v(k)$ với mọi $k=1,2,...,n$.
Gọi ${a_s}= max({a_k})$ $\Rightarrow v(m)= {a_s}$
Giả sử ngoài $s$ ra còn có số $t$ sao cho ${a_t}= {a_s}$ thì viết $t=2^{{a_t}}.{t_1};s=2^{{a_s}}.{s_1}$ trong đó ${t_1},{s_1}$ lẻ.
Suy ra tồn tại số chẵn ${r_1}$ nằm giữa ${s_1}$ và ${t_1}$ $\Rightarrow r= 2^{{a_s}}.{r_1}<t< m\Rightarrow r|m$
Nhưng $r= 2^{{a_s}}.{r_1}\vdots 2^{{a_s}+1}>2^{{a_s}}= v(m)$ suy ra vô lí.
Vậy tồn tại duy nhất số $s$ sao cho ${a_s}= max({a_k})$.Suy ra trong các số $\frac{m}{k}$ chỉ có $\frac{m}{s}$ chẵn.
Suy ra $mS= \frac{m}{1}+\frac{m}{2}+...+\frac{m}{n}$ là lẻ.Mâu thuẫn với ở trên.
$\Rightarrow Q.E.D$
____________________
@hxthanh: $0$ điểm vì chứng minh sai yêu cầu của đề! :))



#385461 CMR IH vuông góc với AM.

Đã gửi bởi Secrets In Inequalities VP on 10-01-2013 - 22:43 trong Hình học


Chả có việc gì .Thêm cách nữa cho nó máu.:)).Mượn hình "Đại ca gin" :))
Dễ thấy tứ giác $BFEC$ nội tiếp đường tròn tâm $M$ có $BF\cap EC= A;EF\cap BC=I;BE\cap CF= H$ nên theo định lí $Brokad$ thì
$M$ là trực tâm tam giác $AHI$ suy ra $IH\perp AM$ suy ra $Q.E.D$



#385450 \[ a^3+b^3+c^3+6\ge (a+b+c)^2 \]

Đã gửi bởi Secrets In Inequalities VP on 10-01-2013 - 22:27 trong Bất đẳng thức và cực trị

$ a,b,c>0:\; abc=1 $ CMR
\[ a^3+b^3+c^3+6\ge (a+b+c)^2 \]

Lâu lâu ghé box BĐT chém tí :)) !
Theo BĐT $Schur$ ta có :
$$a^{3}+b^{3}+c^{3}+3abc\geq ab(a+b)+bc(b+c)+ca(c+a)$$
$$\Rightarrow 3(a^{3}+b^{3}+c^{3})+9abc\geq 3(ab(a+b)+bc(b+c)+ca(c+a))$$
$$\Rightarrow 4(a^{3}+b^{3}+c^{3})+15abc\geq a^{3}+b^{3}+c^{3}+3(ab(a+b)+bc(b+c)+ca(c+a))+6abc$$
$$\Rightarrow 4(a^{3}+b^{3}+c^{3})+15abc\geq (a+b+c)^3$$
$$\Rightarrow 4(a^{3}+b^{3}+c^{3})+15\geq (a+b+c)^3$$
$$\Rightarrow 4(a^{3}+b^{3}+c^{3})+24\geq (a+b+c)^3+9$$
Theo $AM-GM$ thì :
$$\frac{(a+b+c)^3}{3}+\frac{(a+b+c)^3}{3}+ \frac{(a+b+c)^3}{3}+9\geq 4\sqrt[4]{\frac{(a+b+c)^9}{3}}$$
$$ 4\sqrt[4]{\frac{(a+b+c)^9}{3}}\geq 4\sqrt[4]{\frac{(a+b+c)^8.3\sqrt[3]{abc}}{3}}= 4(a+b+c)^2$$
Do đó : $4(a^{3}+b^{3}+c^{3})+24\geq 4(a+b+c)^2$
Chia $2$ vế cho $4$ ta được ngay $Q.E.D$



#385429 { pi + i | 1 ≤ i ≤ n} và { pi - i | 1 ≤ i ≤ n} tạo thành...

Đã gửi bởi Secrets In Inequalities VP on 10-01-2013 - 21:51 trong Số học

Tìm tất cả các số nguyên dương n sao cho tồn tại một hoán vị (p1, p2,..., pn) của các số (1, 2, ..., n) thỏa mãn điều kiện các tập hợp { pi + i | 1 ≤ i ≤ n} và { pi - i | 1 ≤ i ≤ n} tạo thành hệ thặng dư đầy đủ modulo n

Do { pi + i | 1 ≤ i ≤ n} tạo thành hệ thặng dư đầy đủ (HĐĐ) modulo $n$ nên : $\sum_{i=1}^{n}{p_i}+i\equiv \sum_{i=1}^{n}i(Modn)$
Mà do (p1, p2,..., pn) là hoán vị của tập (1, 2, ..., n) nên $\sum_{i=1}^{n}{p_i}+i\equiv 2\sum_{i=1}^{n}i(Modn)$
suy ra $\sum_{i=1}^{n}i\equiv 2\sum_{i=1}^{n}i(Modn)\Rightarrow \sum_{i=1}^{n}i\equiv 0(Modn)\Rightarrow 1+2+...+n\equiv 0(Modn)\Rightarrow \frac{n(n+1)}{2}\equiv 0(Modn)$
Suy ra $n$ phải lẻ .
Do { pi + i | 1 ≤ i ≤ n} cũng tạo thành HĐĐ modulo $n$ nên :$2\sum_{i=1}^{n}i^{2}\equiv \sum_{i=1}^{n}(({p_i}+i)^2+({p_i}-i)^2)\equiv \sum_{i=1}^{n}2{p_i}^{2}+2i^{2}$
Mà do (p1, p2,..., pn) là hoán vị của tập (1, 2, ..., n) nên $\sum_{i=1}^{n}2{p_i}^{2}+2i^{2}\equiv 4\sum_{i=1}^{n}i^{2}(Modn)$
Suy ra $2\sum_{i=1}^{n}i^{2}\equiv 4\sum_{i=1}^{n}i^{2}(Modn)\Rightarrow 2\sum_{i=1}^{n}i^{2}\equiv0(Modn)\Rightarrow 2.(1^{2}+2^{2}+...+n^{2})\equiv 0(Modn)\Rightarrow \frac{n(n+1)(2n+1)}{3}\equiv 0(Modn)$
Suy ra $n$ không chia hết cho 3.
Từ $2$ điều trên ta có : $(n,6)= 1$
Bây giờ ta chứng minh là nếu $(n,6)= 1$ thì tồn tại một hoán vị (p1, p2,..., pn) thỏa mãn bài toán.
Chọn ${p_i}\equiv 2i(Modn)$, ${p_i}\in {1, 2, ..., n}$. Ta có {p1, p2,..., pn} thỏa đề vì $pi + i | 1\leq i\leqn\equiv 3i | 1 \leq i \leq n$ và $pi - i | 1\leq i\leq n \equiv i | 1 \leq i \leq n$ là các hệ thặng dư đầy đủ mô-đun n.



#385404 Chứng minh rằng (2n-3)/5 thoả mãn điều kiện Cn

Đã gửi bởi Secrets In Inequalities VP on 10-01-2013 - 21:10 trong Số học

Cho n là số nguyên dương. Ta nói số nguyên dương k thỏa mãn điều kiện Cn nếu tồn tại 2k số nguyên dương phân biệt a1, b1, ..., ak, bk sao cho a1 + b1, ..., ak+bk cũng phân biệt và nhỏ hơn n.

(a) Chứng minh rằng nếu k thỏa mãn điều kiện Cn thì k <= (2n-3)/5.

(b) Chứng minh rằng 5 thỏa mãn điều kiện C14.

Giả sử rằng (2n-3)/5 nguyên. Chứng minh rằng (2n-3)/5 thoả mãn điều kiện Cn

Đặt $A= \sum_{i=1}^{k}{a_i}+{b_i}$
Vì a1, b1, ..., ak, bk phân biệt nên $A\geq 1+2+...+2k-1+2k= \frac{2k(2k+1)}{2}= k(2k+1)$
Mà do a1 + b1, ..., ak+bk cũng phân biệt và nhỏ hơn n nên :
$A\leq (n-1)+(n-2)+...+(n-(k-1))+(n-k)= kn-\frac{k(k+1)}{2}$
Kết hợp $2$ điều trên ta có : $$kn-\frac{k(k+1)}{2}\geq k(2k+1)$$
$$\Leftrightarrow 2kn-k^2-k\geq 4k^2+2k$$
$$\Leftrightarrow 2kn\geq 5k^2+3k\Leftrightarrow 2n\geq 5k+3$$
$$\Leftrightarrow k\leq \frac{2n-3}{5}$$
$Q.E.D$

b ) Là dấu bằng của câu a.



#385384 Cho $p,q\in N$

Đã gửi bởi Secrets In Inequalities VP on 10-01-2013 - 20:55 trong Số học

Cho $p,q\in N$ sao cho $p+q\in P$ và $n$ chia hết $p^{n}+q^{n}$.Cmr:
$p+q$ chia hết $n$.
Có thể bỏ đk $p+q\in P$ ko? :)

Lâu lâu ms lên ! Thử phát ! :biggrin:
Trường hợp $p+q=2$ dễ thấy đpcm.Xét $p+q\geq 3\Rightarrow p+q$ lẻ suy ra $n$ lẻ
Gọi $k$ là ước nguyên tố nhỏ nhất của $n$.
Gọi $a$ là số thuộc đoạn $[1;k-1]$ sao cho $aq\equiv 1 ( Modk)$ ( số $a$ này luôn tồn tại )
Ta có : $p^{n}+q^{n}\equiv 0(Modn)\Rightarrow p^{n}+q^{n}\equiv 0(Modk)\Rightarrow a^{n}(p^{n}+q^{n})\equiv 0(Modk)$
$\Rightarrow (ap)^n+(aq)^n\equiv 0(Modk)\Rightarrow (ap)^n+1\equiv 0(Modk)$
$\Rightarrow (-ap)^{n}\equiv 1(Modk)$
Gọi $h$ là cấp của $(-ap)$ theo mod $k$ , theo tính chất của cấp suy ra $h|n$ và $h|k-1$
Nếu $h> 1$ thì $h$ sẽ có ít nhất một ước nguyên tố $t$ nào đó
Suy ra $t|n$ và $t|k-1\Rightarrow t\leq k-1< k$ vô lí vì theo định nghĩa số $k$ là ước nguyên tố nhỏ nhất của $n$.
Do đó $h=1$ $\Rightarrow -ap\equiv 1(Modk)\Rightarrow ap+1\vdots k$ .
Lại có $aq\equiv 1(Modk)\Rightarrow aq-1\vdots k\Rightarrow ap+aq\vdots k\Rightarrow a(p+q)\vdots k$
Mà $a\in[1;k-1],p+q\in\mathbb{P}$ suy ra $k= p+q$ $\Rightarrow p+q|n\Rightarrow Q.E.D$

Nếu bỏ $p+q\in\mathbb{P}$ thì chắc là chỉ có $gcd(n,p+q)=t> 1$ thôi :P !



#382453 3 Giao điểm của 3 đường thẳng nằm trên đường tròn nội tiếp tam giác $A_1...

Đã gửi bởi Secrets In Inequalities VP on 01-01-2013 - 09:23 trong Hình học

Hình như bài này là IMO 2000 ^^

@Dark templar:Đừng spam quá em nhé,lần này anh không xóa bài. :closedeyes: